Mathcenter Forum  

Go Back   Mathcenter Forum > คณิตศาสตร์โอลิมปิก และอุดมศึกษา > อสมการ
สมัครสมาชิก คู่มือการใช้ รายชื่อสมาชิก ปฏิทิน ข้อความวันนี้

ตั้งหัวข้อใหม่ Reply
 
เครื่องมือของหัวข้อ ค้นหาในหัวข้อนี้
  #1  
Old 04 กันยายน 2006, 22:58
tani's Avatar
tani tani ไม่อยู่ในระบบ
หัดเดินลมปราณ
 
วันที่สมัครสมาชิก: 10 สิงหาคม 2005
ข้อความ: 42
tani is on a distinguished road
Post ช่วยตรวจสอบให้ผมที

1. เมื่อ a , b , c เป็นจำนวนจริงบวก จงพิสูจน์ว่า
a/(a2+8bc)+b/(b2+8ac)+c/(c2+8ab)1

วิธีคิดของผม

{a/(a2+8bc)+b/(b2+8ac)+c/(c2+8ab)}/3 3abc/(a2+8bc)(b2+8ac)(c2+8ab) ...(โดย A.M.-G.M.) ..... (1)

abc/{(a2+8bc)(b2+8ac)(c2+8ab} มีค่าสูงสุดก็ต่อเมื่อ {(a2+8bc)(b2+8ac)(c2+8ab} มีค่าต่ำสุด

(a2+8bc)(b2+8ac)(c2+8ab 36 18(a2b6c6)(a6b2c6)(a6b6c2) = 33abc ...(โดย A.M.-G.M.)

จาก (1) จะได้ {a/(a2+8bc)+b/(b2+8ac)+c/(c2+8ab)}/3 3abc/(33abc)

\ a/(a2+8bc)+b/(b2+8ac)+c/(c2+8ab) 1 ตามต้องการ

ปล. ที่ผมโพสต์นี้เพื่อที่จะให้ทุกท่านช่วยตรวจสอบวิธีที่ผมทำ ผมไม่แน่ใจว่าสิ่งที่ผมทำนั้นมันถูกรึเปล่าแต่ผมลองคิดดูอย่างหนักแล้วยังไม่พบจุดผิด
แต่ผมคิดว่าน่าจะมีจุดผิดในสิ่งที่ผมทำแต่ผมยังตรวจสอบไม่เจอช่วยด้วยนะครับ
อีกอย่างโจทย์ข้อนี้ผมได้มาจาก หนังสือ สู่เส้นทางอัจฉริยะ (Count Down)
__________________
ความฝันไม่ไกลเกินความพยายาม
ตอบพร้อมอ้างอิงข้อความนี้
  #2  
Old 05 กันยายน 2006, 01:13
nooonuii nooonuii ไม่อยู่ในระบบ
ผู้พิทักษ์กฎทั่วไป
 
วันที่สมัครสมาชิก: 25 พฤษภาคม 2001
ข้อความ: 6,408
nooonuii is on a distinguished road
Post

อ้างอิง:
ข้อความเดิมของคุณ tani:
1. เมื่อ a , b , c เป็นจำนวนจริงบวก จงพิสูจน์ว่า
a/(a2+8bc)+b/(b2+8ac)+c/(c2+8ab)1

วิธีคิดของผม

{a/(a2+8bc)+b/(b2+8ac)+c/(c2+8ab)}/3 3abc/(a2+8bc)(b2+8ac)(c2+8ab) ...(โดย A.M.-G.M.) ..... (1)

abc/{(a2+8bc)(b2+8ac)(c2+8ab} มีค่าสูงสุดก็ต่อเมื่อ {(a2+8bc)(b2+8ac)(c2+8ab} มีค่าต่ำสุด

(a2+8bc)(b2+8ac)(c2+8ab 36 18(a2b6c6)(a6b2c6)(a6b6c2) = 33abc ...(โดย A.M.-G.M.)

จาก (1) จะได้ {a/(a2+8bc)+b/(b2+8ac)+c/(c2+8ab)}/3 3abc/(33abc)

\ a/(a2+8bc)+b/(b2+8ac)+c/(c2+8ab) 1 ตามต้องการ

ปล. ที่ผมโพสต์นี้เพื่อที่จะให้ทุกท่านช่วยตรวจสอบวิธีที่ผมทำ ผมไม่แน่ใจว่าสิ่งที่ผมทำนั้นมันถูกรึเปล่าแต่ผมลองคิดดูอย่างหนักแล้วยังไม่พบจุดผิด
แต่ผมคิดว่าน่าจะมีจุดผิดในสิ่งที่ผมทำแต่ผมยังตรวจสอบไม่เจอช่วยด้วยนะครับ
อีกอย่างโจทย์ข้อนี้ผมได้มาจาก หนังสือ สู่เส้นทางอัจฉริยะ (Count Down)
$\displaystyle{ \frac{a}{\sqrt{a^2+8bc}} + \frac{b}{\sqrt{b^2+8ca}} + \frac{c}{\sqrt{c^2+8ab}} \geq 3\sqrt[3]{ \frac{abc}{\sqrt{a^2+8bc}\sqrt{b^2+8ca}\sqrt{c^2+8ab}} } }$
...(โดย A.M.-G.M.) ..... (1)

$\frac{abc}{\sqrt{a^2+8bc}\sqrt{b^2+8ca}\sqrt{c^2+8ab}} $ มีค่าสูงสุดก็ต่อเมื่อ $\sqrt{a^2+8bc}\sqrt{b^2+8ca}\sqrt{c^2+8ab}$ มีค่าต่ำสุด

^
^
ผิดตั้งแต่บรรทัดนี้แหละครับ จริงๆแล้วเราต้องการค่าต่ำสุดของ $\frac{abc}{\sqrt{a^2+8bc}\sqrt{b^2+8ca}\sqrt{c^2+8ab}} $ ครับ ไม่ใช่ค่าสูงสุด

ป.ล. โจทย์ข้อนี้คือ IMO2001#2 ครับ
__________________
site:mathcenter.net คำค้น

05 กันยายน 2006 09:04 : ข้อความนี้ถูกแก้ไขแล้ว 1 ครั้ง, ครั้งล่าสุดโดยคุณ nooonuii
ตอบพร้อมอ้างอิงข้อความนี้
  #3  
Old 05 กันยายน 2006, 19:17
tani's Avatar
tani tani ไม่อยู่ในระบบ
หัดเดินลมปราณ
 
วันที่สมัครสมาชิก: 10 สิงหาคม 2005
ข้อความ: 42
tani is on a distinguished road
Post

ผมว่าหาค่าสูงสุดนั่นแหละครับถูกต้องแล้ว ยังไงก็ช่วยดูให้อีกทีนะครับ
และผมได้ลองหาค่าสูงสุดมาแล้ว
__________________
ความฝันไม่ไกลเกินความพยายาม
ตอบพร้อมอ้างอิงข้อความนี้
  #4  
Old 05 กันยายน 2006, 19:52
Mastermander's Avatar
Mastermander Mastermander ไม่อยู่ในระบบ
กระบี่ประสานใจ
 
วันที่สมัครสมาชิก: 14 ตุลาคม 2005
ข้อความ: 796
Mastermander is on a distinguished road
Post

ผมว่ามันแปลกๆนะ โดยเฉพาะ A.M.-G.M.
__________________
โลกนี้มีคนอยู่ 10 ประเภท คือ คนที่เข้าใจเลขฐานสอง และคนที่ไม่เข้าใจ
ตอบพร้อมอ้างอิงข้อความนี้
  #5  
Old 06 กันยายน 2006, 04:33
nooonuii nooonuii ไม่อยู่ในระบบ
ผู้พิทักษ์กฎทั่วไป
 
วันที่สมัครสมาชิก: 25 พฤษภาคม 2001
ข้อความ: 6,408
nooonuii is on a distinguished road
Post

เราต้องการ chain ของอสมการแบบนี้ใช่ป่ะครับ

$\displaystyle{ \frac{a}{\sqrt{a^2+8bc}} + \frac{b}{\sqrt{b^2+8ca}} + \frac{c}{\sqrt{c^2+8ab}} \geq 3\sqrt[3]{ \frac{abc}{\sqrt{a^2+8bc}\sqrt{b^2+8ca}\sqrt{c^2+8ab}} }\geq ... \geq ... }$

ถ้าเราพิสูจน์ว่า

$\displaystyle{ \sqrt[3]{ \frac{abc}{\sqrt{a^2+8bc}\sqrt{b^2+8ca}\sqrt{c^2+8ab}} }\leq \frac{1}{3} }$
เราจะได้

$\displaystyle{ \frac{a}{\sqrt{a^2+8bc}} + \frac{b}{\sqrt{b^2+8ca}} + \frac{c}{\sqrt{c^2+8ab}} \geq 3\sqrt[3]{ \frac{abc}{\sqrt{a^2+8bc}\sqrt{b^2+8ca}\sqrt{c^2+8ab}} } \leq 1 }$

แล้วเราจะสรุปอสมการที่ต้องการได้ยังไงครับ
__________________
site:mathcenter.net คำค้น
ตอบพร้อมอ้างอิงข้อความนี้
  #6  
Old 06 กันยายน 2006, 18:50
tani's Avatar
tani tani ไม่อยู่ในระบบ
หัดเดินลมปราณ
 
วันที่สมัครสมาชิก: 10 สิงหาคม 2005
ข้อความ: 42
tani is on a distinguished road
Post

ถ้าผมจะอ้างเหตุผลว่า
อสมการที่ (1) เป็นสมการ (หรือให้ค่าต่ำสุดของพจน์ทางขวามือ)
และอสมการที่ (2) เป็นสมการ (หรือค่าสูงสุดของพจน์ทางซ้ายมือ)
ก็จะได้ตามที่ต้องการคืออสมการในบรรทัดสุดท้าย
ช่วยดูให้อีกทีนะครับ รบกวนด้วยครับสงสัยจริงๆ
__________________
ความฝันไม่ไกลเกินความพยายาม
ตอบพร้อมอ้างอิงข้อความนี้
ตั้งหัวข้อใหม่ Reply



กฎการส่งข้อความ
คุณ ไม่สามารถ ตั้งหัวข้อใหม่ได้
คุณ ไม่สามารถ ตอบหัวข้อได้
คุณ ไม่สามารถ แนบไฟล์และเอกสารได้
คุณ ไม่สามารถ แก้ไขข้อความของคุณเองได้

vB code is On
Smilies are On
[IMG] code is On
HTML code is Off
ทางลัดสู่ห้อง


เวลาที่แสดงทั้งหมด เป็นเวลาที่ประเทศไทย (GMT +7) ขณะนี้เป็นเวลา 06:29


Powered by vBulletin® Copyright ©2000 - 2024, Jelsoft Enterprises Ltd.
Modified by Jetsada Karnpracha